Doctor: In three separate studies, researchers compared children who had slept with night–lights in their rooms as i...

maonuo on August 19, 2020

Please help

I still do not have a good grasp of this question explanation. Could you approach it from another perspective? The only reason I couldn't choose D is because the stimulus mentioned nothing about the quantity of children being studied. So I thought I could eliminate D as an answer.

Replies
Create a free account to read and take part in forum discussions.

Already have an account? log in

Annie on August 20, 2020

Hi @maonuo,

This question asks you to pick the answer choice which most weakens the argument. So, the first step is identifying what the argument is.

Here, the doctor says in 2 studies of older children there was no correlation between night-lights and nearsightedness. In 1 study of younger children there was a correlation between night-lights and nearsightedness. Based on these studies, the doctor concludes that if nearsightedness is caused by night-lights, the effect disappears as the kids get older.

Answer (D) weakens this argument because it questions the accuracy of 2 of the 3 studies the doctor is using to reach his conclusion. If those two studies were not done well, then all we know is that 1 study has shown that there's a correlation between night-lights and nearsightedness, we don't know anything about whether the effect decreases with age.

The answer choice questions the accuracy of the 2 studies by telling us that they didn't examine enough children, aka there weren't enough subjects in the study. You're correct that the stimulus didn't mention this fact. Instead, this answer choice adds in more information about the studies and, by doing so, tells us that they weren't any good.

marinar96 on October 20, 2020

I think what threw me off about this answer choice is that it is not very common to see in a weaken/strengthen question an answer choice that directly and openly attacks specifically one of the premises and not the whole reasoning of the argument (or at least I cannot see how it is attacking the reasoning behind the argument and not just the premise in itself if the sample flaw is not really shown in the stimulus). Could you explain this please? I felt like this answer choice was too simplistic and too direct to be the correct one and I got it wrong because of that so what would you suggest we do when this happens?

lklop on December 23, 2020

^^

Anna20 on February 24, 2021

Yes, please could I follow up on the above too? Thank you!

Anna20 on February 24, 2021

Sorry, to follow up, I picked E because I felt that weakened if 'several of the children who had slept with night lights as children were nearsighted'?

Thank you for all the help!!